Skip to content

Q and A Monday

Daniel R. Grayson edited this page Sep 17, 2019 · 4 revisions

Feel free to ask a question!


Q1. Objects over multiple rings (Yeongrak Kim)

If we define a ring as a local variable inside a method, M2 generates a new base ring at each time. Hence, it is impossible to take further operations on results, unless we define a global variable outside the method.

S=QQ[x,y,z];
M1=random(S^3, S^{3:-1}); 
M2=random(S^3,S^{3:-1}); 
M1+M2 

-- quotient out the last variable 
test=method() 
test(Matrix):= M->( 
    n:=numgens ring M; 
    R:=(coefficientRing S)[t_0..t_(n-2)]; 
    phi:=map(R,S,matrix{{t_0..t_(n-2), 0}}); 
    phi M  
   ) 

-- does not work since their rings are different 
test(M1)+test(M2) 
ring test M1 === ring test M2 

-- The following makes sense, but use global T and psi 
T=(coefficientRing S)[u,v] 
psi=map(T,S,{T_0,T_1,0}) 
psi(M1)+psi(M2) 

What is a better option to implement a method with different base rings as inputs, however, we need to play with several outputs sometimes?

ANSWER: Use "memoize", as shown here, to minimize the number of rings returned:


i2 : help memoize

o2 = memoize -- record results of function evaluation for future use
     ***************************************************************

     Description
     ===========

     memoize f -- produces, from a function f, a new function that behaves the same as f, but remembers
     previous answers to be provided the next time the same arguments are presented.


     +---------------------------------------------------------+
     |i1 : fib = n -> if n <= 1 then 1 else fib(n-1) + fib(n-2)|
     |                                                         |
     |o1 = fib                                                 |
     |                                                         |
     |o1 : FunctionClosure                                     |
     +---------------------------------------------------------+
     |i2 : time fib 28                                         |
     |     -- used 0.362113 seconds                            |
     |                                                         |
     |o2 = 514229                                              |
     +---------------------------------------------------------+
     |i3 : fib = memoize fib                                   |
     |                                                         |
     |o3 = fib                                                 |
     |                                                         |
     |o3 : FunctionClosure                                     |
     +---------------------------------------------------------+
     |i4 : time fib 28                                         |
     |     -- used 0.00003202 seconds                          |
     |                                                         |
     |o4 = 514229                                              |
     +---------------------------------------------------------+
     |i5 : time fib 28                                         |
     |     -- used 1.579e-6 seconds                            |
     |                                                         |
     |o5 = 514229                                              |
     +---------------------------------------------------------+


     The function memoize operates by constructing a "MutableHashTable" in which the argument sequences are
     used as keys for accessing the return value of the function.


     An optional second argument to memoize provides a list of initial values, each of the form x => v, where v
     is the value to be provided for the argument x.


     Warning: when the value returned by f is "null", it will always be recomputed, even if the same arguments
     are presented.


     Warning: the new function created by memoize will save references to all arguments and values it
     encounters, and this will often prevent those arguments and values from being garbage-collected as soon as
     they might have been.  If the arguments are implemented as mutable hash tables (modules, matrices and
     rings are implemented this way) then a viable strategy is to stash computed results in the arguments
     themselves.  See also CacheTable.

     Ways to use memoize :
     =====================

       * memoize(Function)
       * memoize(Function,List)

o2 : DIV

i3 : f = (k,n) -> k[t_0..t_(n-2)]

o3 = f

o3 : FunctionClosure

i4 : f (QQ,3)

o4 = QQ[t , t ]
         0   1

o4 : PolynomialRing

i5 : f (QQ,3)

o5 = QQ[t , t ]
         0   1

o5 : PolynomialRing

i6 : o4_0

o6 = t
      0

o6 : QQ[t , t ]
         0   1

i7 : o5_0

o7 = t
      0

o7 : QQ[t , t ]
         0   1

i8 : o4_0 + o5_0
stdio:8:6:(3): error: expected pair to have a method for '+'

i9 : g = memoize f

o9 = g

o9 : FunctionClosure

i10 : g (QQ,3)

o10 = QQ[t , t ]
          0   1

o10 : PolynomialRing

i11 : g (QQ,3)

o11 = QQ[t , t ]
          0   1

o11 : PolynomialRing

i12 : oo === ooo

o12 = true

i14 : o10_0 + o11_0

o14 = 2t
        0

o14 : QQ[t , t ]
          0   1

i15 : o4 === o5

o15 = false

Q2. (Markus)

Occasionally, some messages refer to files in /Users/dan. Is this intended? For example:

i1 : code assert
stdio:1:1:(3): error: couldn't find file /Users/dan/src/M2/M2.git/M2/Macaulay2/d/startup.m2.in

ANSWER: No, it was not intended. Perhaps it can be fixed, so during the session, I showed how to create an "issue" on github. Here it is: https://github.com/Macaulay2/M2/issues/1005


Q3.

This question concerned how to use y and y_0 simultaneously as variables in rings. The answer was that it is not really possible, since, in order to access y_0, y has to have as its value a table that includes an entry for getting the value of y_0. The discussion led to this code:


i18 : QQ[y]

o18 = QQ[y]

o18 : PolynomialRing

i19 : QQ[y_0]
stdio:19:5:(3): error: no method for binary operator _ applied to objects:
--            y (of class QQ[y])
--      _     0 (of class ZZ)

i20 : y

o20 = y

o20 : QQ[y]

i21 : QQ[symbol y_0]
warning: clearing value of symbol y to allow access to subscripted variables based on it
       : debug with expression   debug 9371   or with command line option   --debug 9371

o21 = QQ[y ]
          0

o21 : PolynomialRing

i22 : y

o22 = y

o22 : IndexedVariableTable

i23 : peek oo

o23 = IndexedVariableTable{0 => y      }
                                 0
                           symbol$ => y

i24 : tt

o24 = tt

o24 : Symbol

i25 : tt = symbol tt

o25 = tt

o25 : Symbol

i26 : local u

o26 = u

o26 : Symbol

i27 : u

i28 : mutable y

o28 = true

i30 : loadPackage "Foo"
--warning: symbol "sin" in Core.Dictionary is shadowed by a symbol in Foo.Dictionary
--  use the synonym Core$sin

o30 = Foo

o30 : Package

i31 : FOO

o31 = FOO

o31 : PolynomialRing

i32 : vars FOO

o32 = | y_0 y_1 |

                1         2
o32 : Matrix FOO  <--- FOO

i33 : y_0

o33 = y
       0

o33 : QQ[y ]
          0

i34 : FOO_0

o34 = y
       0

o34 : FOO

i35 : vars FOO

o35 = | y_0 y_1 |

                1         2
o35 : Matrix FOO  <--- FOO

i36 : y_0 * oo
stdio:36:5:(3): error: no method found for applying promote to:
     argument 1 :  y  (of class QQ[y ])
                    0               0
     argument 2 :  FOO

i37 : degreesRing o21

o37 = ZZ[T]

o37 : PolynomialRing

i38 : T

o38 = T

o38 : Symbol

i39 : use degreesRing o21

o39 = ZZ[T]

o39 : PolynomialRing

i40 : T

o40 = T

o40 : ZZ[T]

i41 : help Variable

o41 = Variable -- specify a name for a variable
      *****************************************

      Description
      ===========

      Variable => x -- an option used with "GF", to specify a symbol to be used as a name for the generator of
      the Galois field.

      Functions with optional argument named Variable :
      =================================================

        * GF(..., Variable => ...), see "GF" -- make a finite field
        * Grassmannian(..., Variable => ...), see "Grassmannian(ZZ,ZZ)" -- the Grassmannian of linear subspaces
          of a vector space
        * "idealizer(..., Variable => ...)" -- Sets the name of the indexed variables introduced in computing
          the endomorphism ring Hom(J,J).
        * "integralClosure(..., Variable => ...)" -- set the base letter for the indexed variables introduced
          while computing the integral closure
        * "integralClosures(..., Variable => ...)"
        * intersectInP(..., Variable => ...), see "intersectInP(..., BasisElementLimit => ...)" -- Option for
          intersectInP
        * multiplicity(..., Variable => ...), see "intersectInP(..., BasisElementLimit => ...)" -- Option for
          intersectInP
        * "makeS2(..., Variable => ...)" -- Sets the name of the indexed variables introduced in computing the
          S2-ification.
        * "ringFromFractions(..., Variable => ...)"
        * Schubert(..., Variable => ...), see "Schubert(ZZ,ZZ,VisibleList)" -- find the Pluecker ideal of a
          Schubert variety
        * distinguished(..., Variable => ...), see "symmetricKernel(..., Variable => ...)" -- Choose name for
          variables in the created ring
        * isReduction(..., Variable => ...), see "symmetricKernel(..., Variable => ...)" -- Choose name for
          variables in the created ring
        * jacobianDual(..., Variable => ...), see "symmetricKernel(..., Variable => ...)" -- Choose name for
          variables in the created ring
        * normalCone(..., Variable => ...), see "symmetricKernel(..., Variable => ...)" -- Choose name for
          variables in the created ring
        * reesAlgebra(..., Variable => ...), see "symmetricKernel(..., Variable => ...)" -- Choose name for
          variables in the created ring
        * reesIdeal(..., Variable => ...), see "symmetricKernel(..., Variable => ...)" -- Choose name for
          variables in the created ring
        * specialFiber(..., Variable => ...), see "symmetricKernel(..., Variable => ...)" -- Choose name for
          variables in the created ring
        * specialFiberIdeal(..., Variable => ...), see "symmetricKernel(..., Variable => ...)" -- Choose name
          for variables in the created ring
        * "symmetricKernel(..., Variable => ...)" -- Choose name for variables in the created ring

      For the programmer
      ==================

      The object "Variable" is a symbol.

o41 : DIV

i42 : help reesIdeal

o42 = reesIdeal -- Compute the defining ideal of the Rees Algebra
      ***********************************************************

      Synopsis
      ========

        * Usage: reesIdeal MreesIdeal(M,f)
        * Inputs:
            * M, a module, or an ideal of a quotient polynomial ring $R$
            * f, a ring element, any non-zerodivisor in ideal or the first Fitting ideal of the module.
              Optional
        * Optional inputs:
            * BasisElementLimit => ..., 
            * DegreeLimit => ...,  -- Bound the degrees considered in the saturation step. Defaults to infinity
            * Jacobian => ..., 
            * MinimalGenerators => ...,  -- Whether the saturation step returns minimal generators
            * PairLimit => ...,  -- Bound the number of s-pairs considered in the saturation step
            * Strategy => ...,  -- Choose a strategy for the saturation step
            * Variable => ..., 
        * Outputs:
            * an ideal, defining the Rees algebra of M

      Description
      ===========

      This routine gives the user a choice between two methods for finding the defining ideal of the Rees
      algebra of an ideal or module $M$ over a ring $R$: The command {\tt reesIdeal(M)} computes a versal
      embedding $g: M\to G$ and a surjection $f: F\to M$ and returns the result of symmetricKernel(gf).

      When M is an ideal (the usual case) or in characteristic 0, the same ideal can be computed by an
      alternate method that is often faster. If the user knows a non-zerodivisor $a\in{} R$ such that
      $M[a^{-1}$ is a free module (for example, when M is an ideal, any non-zerodivisor $a \in{} M$ then it is
      often much faster to compute {\tt reesIdeal(M,a)} which computes the saturation of the defining ideal of
      the symmetric algebra of M with respect to a. This gives the correct answer even under the slightly
      weaker hypothesis that $M[a^{-1}]$ is {\em of linear type}. (See also "isLinearType".)



      +-----------------------------------------------------------------------------+
      |i1 : kk = ZZ/101;                                                            |
      +-----------------------------------------------------------------------------+
      |i2 : S=kk[x_0..x_4];                                                         |
      +-----------------------------------------------------------------------------+
      |i3 : i= trim monomialCurveIdeal(S,{2,3,5,6})                                 |
      |                                                                             |
      |                          2                       3      2     2      2   2  |
      |o3 = ideal (x x  - x x , x  - x x , x x  - x x , x  - x x , x x  - x x , x x |
      |             2 3    1 4   2    0 4   1 2    0 3   3    2 4   1 3    0 4   1 3|
      |     ------------------------------------------------------------------------|
      |                3    2                                                       |
      |     - x x x , x  - x x )                                                    |
      |        0 2 4   1    0 4                                                     |
      |                                                                             |
      |o3 : Ideal of S                                                              |
      +-----------------------------------------------------------------------------+
      |i4 : time V1 = reesIdeal i;                                                  |
      |     -- used 0.0268453 seconds                                               |
      |                                                                             |
      |o4 : Ideal of S[w , w , w , w , w , w , w ]                                  |
      |                 0   1   2   3   4   5   6                                   |
      +-----------------------------------------------------------------------------+
      |i5 : time V2 = reesIdeal(i,i_0);                                             |
      |     -- used 0.0937453 seconds                                               |
      |                                                                             |
      |o5 : Ideal of S[w , w , w , w , w , w , w ]                                  |
      |                 0   1   2   3   4   5   6                                   |
      +-----------------------------------------------------------------------------+

      The following example shows how we handle degrees

      +---------------------------------------------------+
      |i6 : S=kk[a,b,c]                                   |
      |                                                   |
      |o6 = S                                             |
      |                                                   |
      |o6 : PolynomialRing                                |
      +---------------------------------------------------+
      |i7 : m=matrix{{a,0},{b,a},{0,b}}                   |
      |                                                   |
      |o7 = | a 0 |                                       |
      |     | b a |                                       |
      |     | 0 b |                                       |
      |                                                   |
      |             3       2                             |
      |o7 : Matrix S  <--- S                              |
      +---------------------------------------------------+
      |i8 : i=minors(2,m)                                 |
      |                                                   |
      |             2        2                            |
      |o8 = ideal (a , a*b, b )                           |
      |                                                   |
      |o8 : Ideal of S                                    |
      +---------------------------------------------------+
      |i9 : time I1 = reesIdeal i;                        |
      |     -- used 0.0133745 seconds                     |
      |                                                   |
      |o9 : Ideal of S[w , w , w ]                        |
      |                 0   1   2                         |
      +---------------------------------------------------+
      |i10 : time I2 = reesIdeal(i,i_0);                  |
      |     -- used 0.003947 seconds                      |
      |                                                   |
      |o10 : Ideal of S[w , w , w ]                       |
      |                  0   1   2                        |
      +---------------------------------------------------+
      |i11 : transpose gens I1                            |
      |                                                   |
      |o11 = {-1, -3} | aw_1-bw_2    |                    |
      |      {-1, -3} | aw_0-bw_1    |                    |
      |      {-2, -4} | w_1^2-w_0w_2 |                    |
      |                                                   |
      |                            3                     1|
      |o11 : Matrix (S[w , w , w ])  <--- (S[w , w , w ]) |
      |                 0   1   2             0   1   2   |
      +---------------------------------------------------+
      |i12 : transpose gens I2                            |
      |                                                   |
      |o12 = {-1, -3} | aw_1-bw_2    |                    |
      |      {-1, -3} | aw_0-bw_1    |                    |
      |      {-2, -4} | w_1^2-w_0w_2 |                    |
      |                                                   |
      |                            3                     1|
      |o12 : Matrix (S[w , w , w ])  <--- (S[w , w , w ]) |
      |                 0   1   2             0   1   2   |
      +---------------------------------------------------+

      {\bf Investigating plane curve singularities:}

      Proj of the Rees algebra of I \subset{} R is the blowup of I in spec R. Thus the Rees algebra is a basic
      construction in resolution of singularities. Here we work out a simple case:

      +-----------------------------------------------------------------------------------------------------------------------------+
      |i13 : R = ZZ/32003[x,y]                                                                                                      |
      |                                                                                                                             |
      |o13 = R                                                                                                                      |
      |                                                                                                                             |
      |o13 : PolynomialRing                                                                                                         |
      +-----------------------------------------------------------------------------------------------------------------------------+
      |i14 : I = ideal(x,y)                                                                                                         |
      |                                                                                                                             |
      |o14 = ideal (x, y)                                                                                                           |
      |                                                                                                                             |
      |o14 : Ideal of R                                                                                                             |
      +-----------------------------------------------------------------------------------------------------------------------------+
      |i15 : cusp = ideal(x^2-y^3)                                                                                                  |
      |                                                                                                                             |
      |               3    2                                                                                                        |
      |o15 = ideal(- y  + x )                                                                                                       |
      |                                                                                                                             |
      |o15 : Ideal of R                                                                                                             |
      +-----------------------------------------------------------------------------------------------------------------------------+
      |i16 : RI = reesIdeal(I)                                                                                                      |
      |                                                                                                                             |
      |o16 = ideal(x*w  - y*w )                                                                                                     |
      |               0      1                                                                                                      |
      |                                                                                                                             |
      |o16 : Ideal of R[w , w ]                                                                                                     |
      |                  0   1                                                                                                      |
      +-----------------------------------------------------------------------------------------------------------------------------+
      |i17 : S = ring RI                                                                                                            |
      |                                                                                                                             |
      |o17 = S                                                                                                                      |
      |                                                                                                                             |
      |o17 : PolynomialRing                                                                                                         |
      +-----------------------------------------------------------------------------------------------------------------------------+
      |i18 : totalTransform = substitute(cusp, S) + RI                                                                              |
      |                                                                                                                             |
      |                3    2                                                                                                       |
      |o18 = ideal (- y  + x , x*w  - y*w )                                                                                         |
      |                           0      1                                                                                          |
      |                                                                                                                             |
      |o18 : Ideal of S                                                                                                             |
      +-----------------------------------------------------------------------------------------------------------------------------+
      |i19 : D = decompose totalTransform -- the components are the strict transform of the cuspidal curve and the exceptional curve|
      |                                                                                                                             |
      |                            3    2   2              2    2                                                                   |
      |o19 = {ideal (x*w  - y*w , y  - x , y w  - x*w , y*w  - w ), ideal (y, x)}                                                   |
      |                 0      1              0      1     0    1                                                                   |
      |                                                                                                                             |
      |o19 : List                                                                                                                   |
      +-----------------------------------------------------------------------------------------------------------------------------+
      |i20 : totalTransform = first flattenRing totalTransform                                                                      |
      |                                                                                                                             |
      |                3    2                                                                                                       |
      |o20 = ideal (- y  + x , w x - w y)                                                                                           |
      |                         0     1                                                                                             |
      |                                                                                                                             |
      |                 ZZ                                                                                                          |
      |o20 : Ideal of -----[w , w , x, y]                                                                                           |
      |               32003  0   1                                                                                                  |
      +-----------------------------------------------------------------------------------------------------------------------------+
      |i21 : L = primaryDecomposition totalTransform                                                                                |
      |                                                                                                                             |
      |                          3    2     2         2     2           2        2                                                  |
      |o21 = {ideal (w x - w y, y  - x , w y  - w x, w y - w ), ideal (y , x*y, x ,                                                 |
      |               0     1             0      1    0     1                                                                       |
      |      -----------------------------------------------------------------------                                                |
      |      w x - w y)}                                                                                                            |
      |       0     1                                                                                                               |
      |                                                                                                                             |
      |o21 : List                                                                                                                   |
      +-----------------------------------------------------------------------------------------------------------------------------+
      |i22 : apply(L, i -> (degree i)/(degree radical i))                                                                           |
      |                                                                                                                             |
      |o22 = {1, 2}                                                                                                                 |
      |                                                                                                                             |
      |o22 : List                                                                                                                   |
      +-----------------------------------------------------------------------------------------------------------------------------+

      The total transform of the cusp contains the exceptional divisor with multiplicity two.  The strict
      transform of the cusp is a smooth curve but is tangent to the exceptional divisor

      +-------------------------------------------+
      |i23 : use ring L_0                         |
      |                                           |
      |        ZZ                                 |
      |o23 = -----[w , w , x, y]                  |
      |      32003  0   1                         |
      |                                           |
      |o23 : PolynomialRing                       |
      +-------------------------------------------+
      |i24 : singular = ideal(singularLocus(L_0));|
      |                                           |
      |                 ZZ                        |
      |o24 : Ideal of -----[w , w , x, y]         |
      |               32003  0   1                |
      +-------------------------------------------+
      |i25 : SL = saturate(singular, ideal(x,y)); |
      |                                           |
      |                 ZZ                        |
      |o25 : Ideal of -----[w , w , x, y]         |
      |               32003  0   1                |
      +-------------------------------------------+
      |i26 : saturate(SL, ideal(w_0,w_1))         |
      |                                           |
      |o26 = ideal 1                              |
      |                                           |
      |                 ZZ                        |
      |o26 : Ideal of -----[w , w , x, y]         |
      |               32003  0   1                |
      +-------------------------------------------+

      This shows that the strict transform is smooth.

      See also
      ========

        * "symmetricKernel" -- Compute the Rees ring of the image of a matrix
        * "reesAlgebra" -- Compute the defining ideal of the Rees Algebra

      Ways to use reesIdeal :
      =======================

        * reesIdeal(Ideal)
        * reesIdeal(Ideal,RingElement)
        * reesIdeal(Module)
        * reesIdeal(Module,RingElement)

o42 : DIV

i43 : "asd"_0

o43 = a

i44 : class oo

o44 = String

o44 : Type

i45 : QQ["asdf"]

o45 = QQ[asdf]

o45 : PolynomialRing

i46 : asdf

o46 = asdf

o46 : QQ[asdf]

i47 : s_"asdf"

o47 = s
       asdf

o47 : IndexedVariable

i48 : QQ[s_"asdf"]

o48 = QQ[s    ]
          asdf

o48 : PolynomialRing

i49 : "asdf"_"asdf"
stdio:49:7:(3): error: no method for binary operator _ applied to objects:
--            "asdf" (of class String)
--      _     "asdf" (of class String)

i50 : "a" .. "z"

o50 = (a, b, c, d, e, f, g, h, i, j, k, l, m, n, o, p, q, r, s, t, u, v, w, x, y, z)

o50 : Sequence

i51 : QQ[s_"a" .. s_"z"]

o51 = QQ[s , s , s , s , s , s , s , s , s , s , s , s , s , s , s , s , s , s , s , s , s , s , s , s , s , s ]
          a   b   c   d   e   f   g   h   i   j   k   l   m   n   o   p   q   r   s   t   u   v   w   x   y   z

o51 : PolynomialRing

i52 : QQ[s_"aa" .. s_"dd"]

o52 = QQ[s  , s  , s  , s  , s  , s  , s  , s  , s  , s  , s  , s  , s  , s  , s  , s  ]
          aa   ab   ac   ad   ba   bb   bc   bd   ca   cb   cc   cd   da   db   dc   dd

o52 : PolynomialRing

Q4. This question involved the sort order for keys in tables being printed, as here:


i54 : -7 .. 7

o54 = (-7, -6, -5, -4, -3, -2, -1, 0, 1, 2, 3, 4, 5, 6, 7)

o54 : Sequence

i56 : toList oo

o56 = {-7, -6, -5, -4, -3, -2, -1, 0, 1, 2, 3, 4, 5, 6, 7}

o56 : List

i57 : tally oo

o57 = Tally{-1 => 1}
            -2 => 1
            -3 => 1
            -4 => 1
            -5 => 1
            -6 => 1
            -7 => 1
            0 => 1
            1 => 1
            2 => 1
            3 => 1
            4 => 1
            5 => 1
            6 => 1
            7 => 1

o57 : Tally

Notice that the keys in the tally displayed above appear in string-sorting order, not in numerical order. This led to the creation of another issue: https://github.com/Macaulay2/M2/issues/1004


Q5. Please describe describe.

The idea of describe is twofold: to bypass the printing just of the symbol name to which the object has been assigned; and to show more information to the user in mathematician-readable form. (The function peek shows everything inside an object, and is of interest mainly to programmers.) Read about describe at https://faculty.math.illinois.edu/Macaulay2/doc/Macaulay2-1.14/share/doc/Macaulay2/Macaulay2Doc/html/_describe.html .

i64 : viewHelp describe 

i65 : R = QQ[x]

o65 = R

o65 : PolynomialRing

i66 : R

o66 = R

o66 : PolynomialRing

i67 : describe R

o67 = QQ[x, Degrees => {1}, Heft => {1}, MonomialOrder => {MonomialSize => 32}, DegreeRank => 1]
                                                          {GRevLex => {1}    }
                                                          {Position => Up    }

i68 : methods describe

o68 = {0 => (describe, AffineVariety)       }
      {1 => (describe, CoherentSheaf)       }
      {2 => (describe, FractionField)       }
      {3 => (describe, GaloisField)         }
      {4 => (describe, GeneralOrderedMonoid)}
      {5 => (describe, LocalRing)           }
      {6 => (describe, Matrix)              }
      {7 => (describe, Module)              }
      {8 => (describe, PolynomialRing)      }
      {9 => (describe, ProjectiveVariety)   }
      {10 => (describe, QuotientRing)       }
      {11 => (describe, RingMap)            }
      {12 => (describe, Thing)              }

o68 : NumberedVerticalList

Q6. Please talk about about.

The purpose of this function is to search the documentation for entries that match a "regular expression". Read about regular expressions at https://faculty.math.illinois.edu/Macaulay2/doc/Macaulay2-1.14/share/doc/Macaulay2/Macaulay2Doc/html/_regular_spexpressions.html . Read about about at file:///Applications/Macaulay2-1.14/share/doc/Macaulay2/Macaulay2Doc/html/_about.html .


i69 : viewHelp about

i70 : about "resolution"

o70 = {0 => ChainComplexExtras::resolution(ChainComplex)                         }
      {1 => ChainComplexExtras::resolutionOfChainComplex                         }
      {2 => ChainComplexExtras::resolutionOfChainComplex(..., LengthLimit => ...)}
      {3 => ChainComplexExtras::resolutionOfChainComplex(ChainComplex)           }
      {4 => Dmodules::Dresolution                                                }
      {5 => Dmodules::Dresolution(..., LengthLimit => ...)                       }
      {6 => Dmodules::Dresolution(..., Strategy => ...)                          }
      {7 => Dmodules::Dresolution(Ideal)                                         }
      {8 => Dmodules::Dresolution(Ideal,List)                                    }
      {9 => Dmodules::Dresolution(Module)                                        }
      {10 => Dmodules::Dresolution(Module,List)                                  }
      {11 => Macaulay2Doc::computing resolutions                                 }
      {12 => Macaulay2Doc::free resolutions of modules                           }
      {13 => Macaulay2Doc::Hilbert functions and free resolutions                }
      {14 => Macaulay2Doc::resolution                                            }
      {15 => Macaulay2Doc::resolution(..., DegreeLimit => ...)                   }
      {16 => Macaulay2Doc::resolution(..., FastNonminimal => ...)                }
      {17 => Macaulay2Doc::resolution(..., HardDegreeLimit => ...)               }
      {18 => Macaulay2Doc::resolution(..., LengthLimit => ...)                   }
      {19 => Macaulay2Doc::resolution(..., PairLimit => ...)                     }
      {20 => Macaulay2Doc::resolution(..., SortStrategy => ...)                  }
      {21 => Macaulay2Doc::resolution(..., StopBeforeComputation => ...)         }
      {22 => Macaulay2Doc::resolution(..., Strategy => ...)                      }
      {23 => Macaulay2Doc::resolution(..., SyzygyLimit => ...)                   }
      {24 => Macaulay2Doc::resolution(Ideal)                                     }
      {25 => Macaulay2Doc::resolution(Matrix)                                    }
      {26 => Macaulay2Doc::resolution(Module)                                    }
      {27 => Macaulay2Doc::resolution(MonomialIdeal)                             }
      {28 => NCAlgebra::resolution(NCMatrix)                                     }
      {29 => Posets::resolutionPoset                                             }
      {30 => Posets::resolutionPoset(ChainComplex)                               }
      {31 => Posets::resolutionPoset(Ideal)                                      }
      {32 => Posets::resolutionPoset(MonomialIdeal)                              }

o70 : NumberedVerticalList

i71 : about ("resolution", Body => true)

o71 = {0 => BeginningMacaulay2::BeginningMacaulay2                                     }
      {1 => BGG::BGG                                                                   }
      {2 => BGG::directImageComplex(ChainComplex)                                      }
      {3 => BGG::directImageComplex(Matrix)                                            }
      {4 => BGG::pureResolution                                                        }
      {5 => BGG::tateResolution                                                        }
      {6 => BoijSoederberg::BoijSoederberg                                             }
      {7 => BoijSoederberg::facetEquation(List,ZZ,ZZ,ZZ)                               }
      {8 => BoijSoederberg::makePureBetti(List)                                        }
      {9 => BoijSoederberg::pureAll                                                    }
      {10 => BoijSoederberg::pureBetti(List)                                           }
      {11 => BoijSoederberg::pureCharFree                                              }
      {12 => BoijSoederberg::pureTwoInvariant                                          }
      {13 => BoijSoederberg::pureWeyman                                                }
      {14 => BoijSoederberg::randomSocleModule(List,ZZ)                                }
      {15 => Bruns::bruns                                                              }
      {16 => ChainComplexExtras::cartanEilenbergResolution                             }
      {17 => ChainComplexExtras::isExact(..., LengthLimit => ...)                      }
      {18 => ChainComplexExtras::isQuasiIsomorphism(..., LengthLimit => ...)           }
      {19 => ChainComplexExtras::resolution(ChainComplex)                              }
      {20 => ChainComplexExtras::resolutionOfChainComplex                              }
      {21 => ChainComplexExtras::resolutionOfChainComplex(..., LengthLimit => ...)     }
      {22 => ChainComplexExtras::resolutionOfChainComplex(ChainComplex)                }
      {23 => ChainComplexExtras::taylor                                                }
      {24 => ChainComplexExtras::taylorResolution                                      }
      {25 => ChainComplexOperations::ChainComplexOperations                            }
      {26 => CompleteIntersectionResolutions::CompleteIntersectionResolutions          }
      {27 => CompleteIntersectionResolutions::complexity                               }
      {28 => CompleteIntersectionResolutions::EisenbudShamash                          }
      {29 => CompleteIntersectionResolutions::expo                                     }
      {30 => CompleteIntersectionResolutions::exteriorExtModule                        }
      {31 => CompleteIntersectionResolutions::exteriorTorModule                        }
      {32 => CompleteIntersectionResolutions::ExtModuleData                            }
      {33 => CompleteIntersectionResolutions::extVsCohomology                          }
      {34 => CompleteIntersectionResolutions::finiteBettiNumbers                       }
      {35 => CompleteIntersectionResolutions::highSyzygy                               }
      {36 => CompleteIntersectionResolutions::infiniteBettiNumbers                     }
      {37 => CompleteIntersectionResolutions::koszulExtension                          }
      {38 => CompleteIntersectionResolutions::layeredResolution                        }
      {39 => CompleteIntersectionResolutions::makeFiniteResolution                     }
      {40 => CompleteIntersectionResolutions::makeFiniteResolutionCodim2               }
      {41 => CompleteIntersectionResolutions::matrixFactorization                      }
      {42 => CompleteIntersectionResolutions::moduleAsExt                              }
      {43 => CompleteIntersectionResolutions::Shamash                                  }
      {44 => CompleteIntersectionResolutions::TateResolution                           }
      {45 => Complexes::betti(Complex)                                                 }
      {46 => Complexes::cone(ComplexMap)                                               }
      {47 => Complexes::HH Complex                                                     }
      {48 => Complexes::isExact                                                        }
      {49 => Complexes::isFree(Complex)                                                }
      {50 => Complexes::isWellDefined(Complex)                                         }
      {51 => Complexes::length(Complex)                                                }
      {52 => CorrespondenceScrolls::multiHilbertPolynomial                             }
      {53 => DGAlgebras::Basic operations on DG Algebras                               }
      {54 => DGAlgebras::toComplex(DGAlgebra,ZZ)                                       }
      {55 => Dmodules::Ddual                                                           }
      {56 => Dmodules::deRham(..., Strategy => ...)                                    }
      {57 => Dmodules::deRhamAll(..., Strategy => ...)                                 }
      {58 => Dmodules::DExt(..., Strategy => ...)                                      }
      {59 => Dmodules::DHom(..., Strategy => ...)                                      }
      {60 => Dmodules::Dintegration(..., Strategy => ...)                              }
      {61 => Dmodules::DintegrationIdeal(..., Strategy => ...)                         }
      {62 => Dmodules::Dmodules                                                        }
      {63 => Dmodules::Dres                                                            }
      {64 => Dmodules::Dres(..., LengthLimit => ...)                                   }
      {65 => Dmodules::Dres(..., Strategy => ...)                                      }
      {66 => Dmodules::Dresolution                                                     }
      {67 => Dmodules::Dresolution(..., LengthLimit => ...)                            }
      {68 => Dmodules::Dresolution(..., Strategy => ...)                               }
      {69 => Dmodules::Dresolution(Ideal)                                              }
      {70 => Dmodules::Dresolution(Ideal,List)                                         }
      {71 => Dmodules::Dresolution(Module)                                             }
      {72 => Dmodules::Dresolution(Module,List)                                        }
      {73 => Dmodules::Drestriction                                                    }
      {74 => Dmodules::PolyExt(..., Strategy => ...)                                   }
      {75 => Dmodules::RatExt                                                          }
      {76 => Dmodules::RatExt(..., Strategy => ...)                                    }
      {77 => Dmodules::Schreyer                                                        }
      {78 => Dmodules::Vhomogenize                                                     }
      {79 => EdgeIdeals::isChordal                                                     }
      {80 => EdgeIdeals::isSCM                                                         }
      {81 => EdgeIdeals::smallestCycleSize                                             }
      {82 => EliminationMatrices::byResolution                                         }
      {83 => EliminationMatrices::ciResidual                                           }
      {84 => EliminationMatrices::EliminationMatrices                                  }
      {85 => EliminationMatrices::eliminationMatrix                                    }
      {86 => EliminationMatrices::eliminationMatrix(List,Matrix,Matrix)                }
      {87 => EliminationMatrices::regularityVar                                        }
      {88 => FiniteFittingIdeals::co1Fitting                                           }
      {89 => FiniteFittingIdeals::nextDegree                                           }
      {90 => FourierMotzkin::Applications to multigraded polynomial rings              }
      {91 => HigherCIOperators::ciOperatorResolution                                   }
      {92 => HigherCIOperators::HigherCIOperators                                      }
      {93 => HigherCIOperators::higherCIOperators                                      }
      {94 => HigherCIOperators::makeALDifferential                                     }
      {95 => HighestWeights::Example 1                                                 }
      {96 => HighestWeights::Example 2                                                 }
      {97 => HighestWeights::Example 3                                                 }
      {98 => HighestWeights::Example 4                                                 }
      {99 => HighestWeights::Example 5                                                 }
      {100 => HighestWeights::Example 6                                                }
      {101 => HighestWeights::Example 7                                                }
      {102 => HighestWeights::HighestWeights                                           }
      {103 => HighestWeights::highestWeightsDecomposition(ChainComplex,ZZ,List)        }
      {104 => HighestWeights::propagateWeights                                         }
      {105 => HighestWeights::propagateWeights(Matrix,List)                            }
      {106 => HyperplaneArrangements::EPY                                              }
      {107 => InvolutiveBases::Involutive                                              }
      {108 => InvolutiveBases::InvolutiveBases                                         }
      {109 => InvolutiveBases::janetBasis                                              }
      {110 => InvolutiveBases::janetResolution                                         }
      {111 => InvolutiveBases::multVar                                                 }
      {112 => InvolutiveBases::multVars                                                }
      {113 => K3Carpets::allGradings                                                   }
      {114 => K3Carpets::analyzeStrand                                                 }
      {115 => K3Carpets::canonicalHomotopies                                           }
      {116 => K3Carpets::carpetBettiTable                                              }
      {117 => K3Carpets::carpetBettiTables                                             }
      {118 => K3Carpets::carpetDet                                                     }
      {119 => K3Carpets::computeBound                                                  }
      {120 => K3Carpets::degenerateK3BettiTables                                       }
      {121 => K3Carpets::gorensteinDouble                                              }
      {122 => K3Carpets::K3Carpets                                                     }
      {123 => K3Carpets::relativeResolution                                            }
      {124 => K3Carpets::relativeResolutionTwists                                      }
      {125 => K3Carpets::resonanceDet                                                  }
      {126 => K3Carpets::schreyerName                                                  }
      {127 => KustinMiller::Cyclic Polytopes                                           }
      {128 => KustinMiller::isExactRes                                                 }
      {129 => KustinMiller::Jerry                                                      }
      {130 => KustinMiller::KustinMiller                                               }
      {131 => KustinMiller::kustinMillerComplex                                        }
      {132 => KustinMiller::resBE                                                      }
      {133 => KustinMiller::Stellar Subdivisions                                       }
      {134 => KustinMiller::Tom                                                        }
      {135 => LexIdeals::cancelAll                                                     }
      {136 => LexIdeals::LPP                                                           }
      {137 => LexIdeals::multBounds                                                    }
      {138 => LexIdeals::multLowerBound                                                }
      {139 => LexIdeals::multUpperBound                                                }
      {140 => LexIdeals::multUpperHF                                                   }
      {141 => LocalRings::LocalRings                                                   }
      {142 => Macaulay2Doc::a first Macaulay2 session                                  }
      {143 => Macaulay2Doc::betti                                                      }
      {144 => Macaulay2Doc::betti(..., Minimize => ...)                                }
      {145 => Macaulay2Doc::betti(BettiTally)                                          }
      {146 => Macaulay2Doc::betti(GradedModule)                                        }
      {147 => Macaulay2Doc::betti(GroebnerBasis)                                       }
      {148 => Macaulay2Doc::betti(Ideal)                                               }
      {149 => Macaulay2Doc::betti(Matrix)                                              }
      {150 => Macaulay2Doc::betti(Module)                                              }
      {151 => Macaulay2Doc::chain complexes                                            }
      {152 => Macaulay2Doc::ChainComplex                                               }
      {153 => Macaulay2Doc::ChainComplex ++ ChainComplex                               }
      {154 => Macaulay2Doc::ChainComplex _ ZZ                                          }
      {155 => Macaulay2Doc::ChainComplexMap _ ZZ                                       }
      {156 => Macaulay2Doc::changes, 1.0 and 1.1                                       }
      {157 => Macaulay2Doc::changes, 1.6                                               }
      {158 => Macaulay2Doc::changes, 1.7                                               }
      {159 => Macaulay2Doc::changes, 1.8                                               }
      {160 => Macaulay2Doc::changes, 1.9                                               }
      {161 => Macaulay2Doc::changes, 1.9.1                                             }
      {162 => Macaulay2Doc::changes, 1.10                                              }
      {163 => Macaulay2Doc::changes, 1.11                                              }
      {164 => Macaulay2Doc::changes, 1.12                                              }
      {165 => Macaulay2Doc::changes, 1.13                                              }
      {166 => Macaulay2Doc::complete                                                   }
      {167 => Macaulay2Doc::complete(ChainComplex)                                     }
      {168 => Macaulay2Doc::computing resolutions                                      }
      {169 => Macaulay2Doc::cone(ChainComplexMap)                                      }
      {170 => Macaulay2Doc::David Eisenbud                                             }
      {171 => Macaulay2Doc::dd                                                         }
      {172 => Macaulay2Doc::document                                                   }
      {173 => Macaulay2Doc::document(..., Inputs => ...)                               }
      {174 => Macaulay2Doc::document(..., Key => ...)                                  }
      {175 => Macaulay2Doc::document(..., Outputs => ...)                              }
      {176 => Macaulay2Doc::documentation keys                                         }
      {177 => Macaulay2Doc::eagonNorthcott(Matrix)                                     }
      {178 => Macaulay2Doc::Elementary uses of Groebner bases I. Math 634 Fall 2005    }
      {179 => Macaulay2Doc::engine                                                     }
      {180 => Macaulay2Doc::Ext^ZZ(CoherentSheaf,CoherentSheaf)                        }
      {181 => Macaulay2Doc::Ext^ZZ(CoherentSheaf,SumOfTwists)                          }
      {182 => Macaulay2Doc::Ext^ZZ(Matrix,Module)                                      }
      {183 => Macaulay2Doc::Ext^ZZ(Module,Matrix)                                      }
      {184 => Macaulay2Doc::Ext^ZZ(Module,Module)                                      }
      {185 => Macaulay2Doc::extend(ChainComplex,ChainComplex,Matrix)                   }
      {186 => Macaulay2Doc::extracting information from chain complexes                }
      {187 => Macaulay2Doc::factor                                                     }
      {188 => Macaulay2Doc::FastNonminimal                                             }
      {189 => Macaulay2Doc::findSynonyms(Symbol)                                       }
      {190 => Macaulay2Doc::frac                                                       }
      {191 => Macaulay2Doc::free resolutions of modules                                }
      {192 => Macaulay2Doc::gbTrace                                                    }
      {193 => Macaulay2Doc::getNonUnit                                                 }
      {194 => Macaulay2Doc::graded and multigraded polynomial rings                    }
      {195 => Macaulay2Doc::heft                                                       }
      {196 => Macaulay2Doc::HH^ZZ CoherentSheaf                                        }
      {197 => Macaulay2Doc::Hilbert functions and free resolutions                     }
      {198 => Macaulay2Doc::Hom(Module,ChainComplex)                                   }
      {199 => Macaulay2Doc::Ideal                                                      }
      {200 => Macaulay2Doc::ideals                                                     }
      {201 => Macaulay2Doc::ideals to and from modules                                 }
      {202 => Macaulay2Doc::LengthLimit                                                }
      {203 => Macaulay2Doc::lift                                                       }
      {204 => Macaulay2Doc::maps between chain complexes                               }
      {205 => Macaulay2Doc::methods                                                    }
      {206 => Macaulay2Doc::minimalBetti(Ideal)                                        }
      {207 => Macaulay2Doc::Module                                                     }
      {208 => Macaulay2Doc::modules                                                    }
      {209 => Macaulay2Doc::modules in Macaulay2                                       }
      {210 => Macaulay2Doc::nullhomotopy                                               }
      {211 => Macaulay2Doc::packages provided with Macaulay2                           }
      {212 => Macaulay2Doc::path                                                       }
      {213 => Macaulay2Doc::pdim                                                       }
      {214 => Macaulay2Doc::peek'(ZZ,Thing)                                            }
      {215 => Macaulay2Doc::pseudocode                                                 }
      {216 => Macaulay2Doc::regularity                                                 }
      {217 => Macaulay2Doc::replacements for commands and scripts from Macaulay        }
      {218 => Macaulay2Doc::Resolution                                                 }
      {219 => Macaulay2Doc::resolution                                                 }
      {220 => Macaulay2Doc::resolution(..., DegreeLimit => ...)                        }
      {221 => Macaulay2Doc::resolution(..., FastNonminimal => ...)                     }
      {222 => Macaulay2Doc::resolution(..., HardDegreeLimit => ...)                    }
      {223 => Macaulay2Doc::resolution(..., LengthLimit => ...)                        }
      {224 => Macaulay2Doc::resolution(..., PairLimit => ...)                          }
      {225 => Macaulay2Doc::resolution(..., SortStrategy => ...)                       }
      {226 => Macaulay2Doc::resolution(..., StopBeforeComputation => ...)              }
      {227 => Macaulay2Doc::resolution(..., Strategy => ...)                           }
      {228 => Macaulay2Doc::resolution(..., SyzygyLimit => ...)                        }
      {229 => Macaulay2Doc::resolution(Ideal)                                          }
      {230 => Macaulay2Doc::resolution(Matrix)                                         }
      {231 => Macaulay2Doc::resolution(Module)                                         }
      {232 => Macaulay2Doc::resolution(MonomialIdeal)                                  }
      {233 => Macaulay2Doc::RingMap RingElement                                        }
      {234 => Macaulay2Doc::running Macaulay2 in emacs                                 }
      {235 => Macaulay2Doc::Schreyer orders                                            }
      {236 => Macaulay2Doc::Singular Book 2.4.12                                       }
      {237 => Macaulay2Doc::Singular Book 2.4.15                                       }
      {238 => Macaulay2Doc::Singular Book 2.5.18                                       }
      {239 => Macaulay2Doc::source(ChainComplexMap)                                    }
      {240 => Macaulay2Doc::status                                                     }
      {241 => Macaulay2Doc::syzygyScheme                                               }
      {242 => Macaulay2Doc::target(ChainComplexMap)                                    }
      {243 => Macaulay2Doc::Tutorial: Canonical Embeddings of Plane Curves and Gonality}
      {244 => Macaulay2Doc::Tutorial: Elementary uses of Groebner bases                }
      {245 => Macaulay2Doc::Tutorial: Fano varieties                                   }
      {246 => Macaulay2Doc::two dimensional formatting                                 }
      {247 => Macaulay2Doc::Usage                                                      }
      {248 => Macaulay2Doc::using Macaulay2 with emacs after it has been set up        }
      {249 => MCMApproximations::approximation                                         }
      {250 => MCMApproximations::MCMApproximations                                     }
      {251 => MCMApproximations::syzygyModule                                          }
      {252 => MultiplierIdealsDim2::MultiplierIdealsDim2                               }
      {253 => MultiplierIdealsDim2::RelativeCanonicalDivisor                           }
      {254 => NCAlgebra::Basic operations on noncommutative algebras                   }
      {255 => NCAlgebra::betti(NCChainComplex)                                         }
      {256 => NCAlgebra::NCChainComplex                                                }
      {257 => NCAlgebra::NCMatrix // NCMatrix                                          }
      {258 => NCAlgebra::resolution(NCMatrix)                                          }
      {259 => NCAlgebra::rightKernel                                                   }
      {260 => NCAlgebra::Using the Bergman interface                                   }
      {261 => NonminimalComplexes::SVDBetti                                            }
      {262 => NormalToricVarieties::makeSmooth(NormalToricVariety)                     }
      {263 => NormalToricVarieties::Making normal toric varieties                      }
      {264 => NormalToricVarieties::Resolution of singularities                        }
      {265 => PieriMaps::pieri                                                         }
      {266 => PieriMaps::PieriMaps                                                     }
      {267 => PieriMaps::pureFree                                                      }
      {268 => Points::expectedBetti                                                    }
      {269 => Points::minMaxResolution                                                 }
      {270 => Points::Points                                                           }
      {271 => Posets::Example: Constructing common posets                              }
      {272 => Posets::Example: Hibi ideals                                             }
      {273 => Posets::Example: LCM-lattices                                            }
      {274 => Posets::lcmLattice                                                       }
      {275 => Posets::resolutionPoset                                                  }
      {276 => Posets::resolutionPoset(ChainComplex)                                    }
      {277 => Posets::resolutionPoset(Ideal)                                           }
      {278 => Posets::resolutionPoset(MonomialIdeal)                                   }
      {279 => PrimaryDecomposition::primaryDecomposition(..., Strategy => ...)         }
      {280 => PruneComplex::PruneComplex                                               }
      {281 => PruneComplex::pruneComplex                                               }
      {282 => PruneComplex::toChainComplex                                             }
      {283 => RandomMonomialIdeals::bettiStats                                         }
      {284 => RandomSpaceCurves::knownUnirationalComponentOfSpaceCurves                }
      {285 => RandomSpaceCurves::spaceCurve                                            }
      {286 => ReesAlgebra::PlaneCurveSingularities                                     }
      {287 => ReesAlgebra::ReesAlgebra                                                 }
      {288 => ReesAlgebra::reesIdeal                                                   }
      {289 => Regularity::Regularity                                                   }
      {290 => RelativeCanonicalResolution::eagonNorthcottType                          }
      {291 => RelativeCanonicalResolution::iteratedMC                                  }
      {292 => RelativeCanonicalResolution::liftMatrixToENT                             }
      {293 => RelativeCanonicalResolution::RelativeCanonicalResolution                 }
      {294 => RelativeCanonicalResolution::resCurveOnScroll                            }
      {295 => RelativeCanonicalResolution::rkSyzModules                                }
      {296 => SchurRings::schurResolution                                              }
      {297 => SchurRings::schurResolution(..., DegreeLimit => ...)                     }
      {298 => SchurRings::schurResolution(..., SyzygyLimit => ...)                     }
      {299 => SchurRings::SchurRings                                                   }
      {300 => SimplicialComplexes::lyubeznikComplex                                    }
      {301 => SimplicialComplexes::superficialComplex                                  }
      {302 => SLnEquivariantMatrices::SLnEquivariantMatrices                           }
      {303 => SpectralSequences::basis(List,SpectralSequencePage)                      }
      {304 => SpectralSequences::hilbertPolynomial(SpectralSequencePage)               }
      {305 => SpectralSequences::minimalPresentation(SpectralSequence)                 }
      {306 => SpectralSequences::minimalPresentation(SpectralSequencePage)             }
      {307 => SpectralSequences::Page                                                  }
      {308 => SpectralSequences::PageMap                                               }
      {309 => SpectralSequences::pruningMaps                                           }
      {310 => SpectralSequences::pruningMaps(SpectralSequencePage)                     }
      {311 => SpectralSequences::Spectral sequences and hypercohomology calculations   }
      {312 => SpectralSequences::SpectralSequence ^ ZZ                                 }
      {313 => SpectralSequences::SpectralSequence _ ZZ                                 }
      {314 => SpectralSequences::spectralSequencePage(FilteredComplex,ZZ)              }
      {315 => SpectralSequences::SpectralSequencePageMap ^ List                        }
      {316 => SpectralSequences::SpectralSequencePageMap _ List                        }
      {317 => TateOnProducts::beilinson                                                }
      {318 => TateOnProducts::beilinsonWindow                                          }
      {319 => TateOnProducts::coarseMultigradedRegularity                              }
      {320 => TateOnProducts::cohomologyHashTable                                      }
      {321 => TateOnProducts::cohomologyMatrix                                         }
      {322 => TateOnProducts::composedFunctions                                        }
      {323 => TateOnProducts::cornerComplex                                            }
      {324 => TateOnProducts::eulerPolynomialTable                                     }
      {325 => TateOnProducts::firstQuadrantComplex                                     }
      {326 => TateOnProducts::lastQuadrantComplex                                      }
      {327 => TateOnProducts::strand                                                   }
      {328 => TateOnProducts::tateExtension                                            }
      {329 => TateOnProducts::TateOnProducts                                           }
      {330 => TateOnProducts::tateResolution                                           }
      {331 => TensorComplexes::flattenedESTensor                                       }
      {332 => TensorComplexes::pureResES                                               }
      {333 => TensorComplexes::pureResES1                                              }
      {334 => TensorComplexes::pureResTC                                               }
      {335 => TensorComplexes::pureResTC1                                              }
      {336 => TensorComplexes::tensorComplex1                                          }
      {337 => TensorComplexes::TensorComplexes                                         }
      {338 => TestIdeals::isCohenMacaulay                                              }
      {339 => Text::html(TEX)                                                          }
      {340 => Text::MarkUpTypeWithOptions                                              }
      {341 => TorAlgebra::TorAlgebra                                                   }
      {342 => Triplets::Triplet                                                        }
      {343 => Triplets::Triplets                                                       }
      {344 => VectorFields::isFreeDivisor                                              }
      {345 => VersalDeformations::versalDeformation                                    }
      {346 => VirtualResolutions::isVirtual                                            }
      {347 => VirtualResolutions::isVirtual(..., Strategy => ...)                      }
      {348 => VirtualResolutions::resolveViaFatPoint                                   }
      {349 => VirtualResolutions::virtualOfPair                                        }
      {350 => VirtualResolutions::VirtualResolutions                                   }

o71 : NumberedVerticalList

i72 : help 341

o72 = TorAlgebra::TorAlgebra -- Classification of local rings based on multiplication in homology
      *******************************************************************************************

      Let $I$ be an ideal of a regular local ring $Q$ with residue field $k$. The minimal free resolution of
      $R=Q/I$ carries a structure of a differential graded algebra. If the length of the resolution, which is
      called the codepth of $R$, is at most $3$, then the induced algebra structure on Tor$_Q*$ ($R,k$) is
      unique and provides for a classification of such local rings.

      According to the multiplicative structure on Tor$_Q*$ ($R,k$), a non-zero local ring $R$ of codepth at
      most 3 belongs to exactly one of the (parametrized) classes designated {\bf B}, {\bf C}(c), {\bf G}(r),
      {\bf H}(p,q), {\bf S}, or {\bf T}. An overview of the theory can be found in L.L. Avramov, {\it A
      cohomological study of local rings of embedding codepth 3}, http://arxiv.org/abs/1105.3991.

      There is a similar classification of Gorenstein local rings of codepth 4, due to A.R. Kustin and M.
      Miller. There are four classes, which in the original paper, {\it Classification of the Tor-Algebras of
      Codimension Four Gorenstein Local rings} https://doi.org/10.1007/BF01215134, are called A, B, C, and D,
      while in the survey {\it Homological asymptotics of modules over local rings}
      https://doi.org/10.1007/978-1-4612-3660-3_3 by L.L. Avramov, they are called CI, GGO, GTE, and GH(p),
      respectively. Here we denote these classes {\bf C}(c), {\bf GS}, {\bf GT}, and {\bf GH}(p), respectively.

      The package implements an algorithm for classification of local rings in the sense discussed above. For
      rings of codepth at most 3 it is described in L.W. Christensen and O. Veliche, {\it Local rings of
      embedding codepth 3: a classification algorithm}, http://arxiv.org/abs/1402.4052. The classification of
      Gorenstein rings of codepth 4 is analogous.

      The package also recognizes Golod rings, Gorenstein rings, and complete intersection rings of any
      codepth. To recognize Golod rings the package implements a test found in J. Burke, {\it Higher homotopies
      and Golod rings} https://arxiv.org/abs/1508.03782.

o72 : DIV

i73 : about ("res...tion", Body => true)

o73 = {0 => BeginningMacaulay2::BeginningMacaulay2                                     }
      {1 => BGG::BGG                                                                   }
      {2 => BGG::directImageComplex(ChainComplex)                                      }
      {3 => BGG::directImageComplex(Matrix)                                            }
      {4 => BGG::pureResolution                                                        }
      {5 => BGG::tateResolution                                                        }
      {6 => BoijSoederberg::BoijSoederberg                                             }
      {7 => BoijSoederberg::facetEquation(List,ZZ,ZZ,ZZ)                               }
      {8 => BoijSoederberg::makePureBetti(List)                                        }
      {9 => BoijSoederberg::pureAll                                                    }
      {10 => BoijSoederberg::pureBetti(List)                                           }
      {11 => BoijSoederberg::pureCharFree                                              }
      {12 => BoijSoederberg::pureTwoInvariant                                          }
      {13 => BoijSoederberg::pureWeyman                                                }
      {14 => BoijSoederberg::randomSocleModule(List,ZZ)                                }
      {15 => Bruns::bruns                                                              }
      {16 => ChainComplexExtras::cartanEilenbergResolution                             }
      {17 => ChainComplexExtras::isExact(..., LengthLimit => ...)                      }
      {18 => ChainComplexExtras::isQuasiIsomorphism(..., LengthLimit => ...)           }
      {19 => ChainComplexExtras::resolution(ChainComplex)                              }
      {20 => ChainComplexExtras::resolutionOfChainComplex                              }
      {21 => ChainComplexExtras::resolutionOfChainComplex(..., LengthLimit => ...)     }
      {22 => ChainComplexExtras::resolutionOfChainComplex(ChainComplex)                }
      {23 => ChainComplexExtras::taylor                                                }
      {24 => ChainComplexExtras::taylorResolution                                      }
      {25 => ChainComplexOperations::ChainComplexOperations                            }
      {26 => CompleteIntersectionResolutions::CompleteIntersectionResolutions          }
      {27 => CompleteIntersectionResolutions::complexity                               }
      {28 => CompleteIntersectionResolutions::EisenbudShamash                          }
      {29 => CompleteIntersectionResolutions::expo                                     }
      {30 => CompleteIntersectionResolutions::exteriorExtModule                        }
      {31 => CompleteIntersectionResolutions::exteriorTorModule                        }
      {32 => CompleteIntersectionResolutions::ExtModuleData                            }
      {33 => CompleteIntersectionResolutions::extVsCohomology                          }
      {34 => CompleteIntersectionResolutions::finiteBettiNumbers                       }
      {35 => CompleteIntersectionResolutions::highSyzygy                               }
      {36 => CompleteIntersectionResolutions::infiniteBettiNumbers                     }
      {37 => CompleteIntersectionResolutions::koszulExtension                          }
      {38 => CompleteIntersectionResolutions::layeredResolution                        }
      {39 => CompleteIntersectionResolutions::makeFiniteResolution                     }
      {40 => CompleteIntersectionResolutions::makeFiniteResolutionCodim2               }
      {41 => CompleteIntersectionResolutions::matrixFactorization                      }
      {42 => CompleteIntersectionResolutions::moduleAsExt                              }
      {43 => CompleteIntersectionResolutions::Shamash                                  }
      {44 => CompleteIntersectionResolutions::TateResolution                           }
      {45 => Complexes::betti(Complex)                                                 }
      {46 => Complexes::cone(ComplexMap)                                               }
      {47 => Complexes::HH Complex                                                     }
      {48 => Complexes::isExact                                                        }
      {49 => Complexes::isFree(Complex)                                                }
      {50 => Complexes::isWellDefined(Complex)                                         }
      {51 => Complexes::length(Complex)                                                }
      {52 => CorrespondenceScrolls::multiHilbertPolynomial                             }
      {53 => DGAlgebras::Basic operations on DG Algebras                               }
      {54 => DGAlgebras::toComplex(DGAlgebra,ZZ)                                       }
      {55 => Dmodules::Ddual                                                           }
      {56 => Dmodules::deRham(..., Strategy => ...)                                    }
      {57 => Dmodules::deRhamAll(..., Strategy => ...)                                 }
      {58 => Dmodules::DExt(..., Strategy => ...)                                      }
      {59 => Dmodules::DHom(..., Strategy => ...)                                      }
      {60 => Dmodules::Dintegration(..., Strategy => ...)                              }
      {61 => Dmodules::DintegrationIdeal(..., Strategy => ...)                         }
      {62 => Dmodules::Dmodules                                                        }
      {63 => Dmodules::Dres                                                            }
      {64 => Dmodules::Dres(..., LengthLimit => ...)                                   }
      {65 => Dmodules::Dres(..., Strategy => ...)                                      }
      {66 => Dmodules::Dresolution                                                     }
      {67 => Dmodules::Dresolution(..., LengthLimit => ...)                            }
      {68 => Dmodules::Dresolution(..., Strategy => ...)                               }
      {69 => Dmodules::Dresolution(Ideal)                                              }
      {70 => Dmodules::Dresolution(Ideal,List)                                         }
      {71 => Dmodules::Dresolution(Module)                                             }
      {72 => Dmodules::Dresolution(Module,List)                                        }
      {73 => Dmodules::Drestriction                                                    }
      {74 => Dmodules::PolyExt(..., Strategy => ...)                                   }
      {75 => Dmodules::RatExt                                                          }
      {76 => Dmodules::RatExt(..., Strategy => ...)                                    }
      {77 => Dmodules::Schreyer                                                        }
      {78 => Dmodules::Vhomogenize                                                     }
      {79 => EdgeIdeals::isChordal                                                     }
      {80 => EdgeIdeals::isSCM                                                         }
      {81 => EdgeIdeals::smallestCycleSize                                             }
      {82 => EliminationMatrices::byResolution                                         }
      {83 => EliminationMatrices::ciResidual                                           }
      {84 => EliminationMatrices::EliminationMatrices                                  }
      {85 => EliminationMatrices::eliminationMatrix                                    }
      {86 => EliminationMatrices::eliminationMatrix(List,Matrix,Matrix)                }
      {87 => EliminationMatrices::regularityVar                                        }
      {88 => FiniteFittingIdeals::co1Fitting                                           }
      {89 => FiniteFittingIdeals::nextDegree                                           }
      {90 => FourierMotzkin::Applications to multigraded polynomial rings              }
      {91 => HigherCIOperators::ciOperatorResolution                                   }
      {92 => HigherCIOperators::HigherCIOperators                                      }
      {93 => HigherCIOperators::higherCIOperators                                      }
      {94 => HigherCIOperators::makeALDifferential                                     }
      {95 => HighestWeights::Example 1                                                 }
      {96 => HighestWeights::Example 2                                                 }
      {97 => HighestWeights::Example 3                                                 }
      {98 => HighestWeights::Example 4                                                 }
      {99 => HighestWeights::Example 5                                                 }
      {100 => HighestWeights::Example 6                                                }
      {101 => HighestWeights::Example 7                                                }
      {102 => HighestWeights::HighestWeights                                           }
      {103 => HighestWeights::highestWeightsDecomposition(ChainComplex,ZZ,List)        }
      {104 => HighestWeights::propagateWeights                                         }
      {105 => HighestWeights::propagateWeights(Matrix,List)                            }
      {106 => HyperplaneArrangements::EPY                                              }
      {107 => InvolutiveBases::Involutive                                              }
      {108 => InvolutiveBases::InvolutiveBases                                         }
      {109 => InvolutiveBases::janetBasis                                              }
      {110 => InvolutiveBases::janetResolution                                         }
      {111 => InvolutiveBases::multVar                                                 }
      {112 => InvolutiveBases::multVars                                                }
      {113 => K3Carpets::allGradings                                                   }
      {114 => K3Carpets::analyzeStrand                                                 }
      {115 => K3Carpets::canonicalHomotopies                                           }
      {116 => K3Carpets::carpetBettiTable                                              }
      {117 => K3Carpets::carpetBettiTables                                             }
      {118 => K3Carpets::carpetDet                                                     }
      {119 => K3Carpets::computeBound                                                  }
      {120 => K3Carpets::degenerateK3BettiTables                                       }
      {121 => K3Carpets::gorensteinDouble                                              }
      {122 => K3Carpets::K3Carpets                                                     }
      {123 => K3Carpets::relativeResolution                                            }
      {124 => K3Carpets::relativeResolutionTwists                                      }
      {125 => K3Carpets::resonanceDet                                                  }
      {126 => K3Carpets::schreyerName                                                  }
      {127 => KustinMiller::Cyclic Polytopes                                           }
      {128 => KustinMiller::isExactRes                                                 }
      {129 => KustinMiller::Jerry                                                      }
      {130 => KustinMiller::KustinMiller                                               }
      {131 => KustinMiller::kustinMillerComplex                                        }
      {132 => KustinMiller::resBE                                                      }
      {133 => KustinMiller::Stellar Subdivisions                                       }
      {134 => KustinMiller::Tom                                                        }
      {135 => LexIdeals::cancelAll                                                     }
      {136 => LexIdeals::LPP                                                           }
      {137 => LexIdeals::multBounds                                                    }
      {138 => LexIdeals::multLowerBound                                                }
      {139 => LexIdeals::multUpperBound                                                }
      {140 => LexIdeals::multUpperHF                                                   }
      {141 => LocalRings::LocalRings                                                   }
      {142 => Macaulay2Doc::a first Macaulay2 session                                  }
      {143 => Macaulay2Doc::betti                                                      }
      {144 => Macaulay2Doc::betti(..., Minimize => ...)                                }
      {145 => Macaulay2Doc::betti(BettiTally)                                          }
      {146 => Macaulay2Doc::betti(GradedModule)                                        }
      {147 => Macaulay2Doc::betti(GroebnerBasis)                                       }
      {148 => Macaulay2Doc::betti(Ideal)                                               }
      {149 => Macaulay2Doc::betti(Matrix)                                              }
      {150 => Macaulay2Doc::betti(Module)                                              }
      {151 => Macaulay2Doc::chain complexes                                            }
      {152 => Macaulay2Doc::ChainComplex                                               }
      {153 => Macaulay2Doc::ChainComplex ++ ChainComplex                               }
      {154 => Macaulay2Doc::ChainComplex _ ZZ                                          }
      {155 => Macaulay2Doc::ChainComplexMap _ ZZ                                       }
      {156 => Macaulay2Doc::changes, 1.0 and 1.1                                       }
      {157 => Macaulay2Doc::changes, 1.6                                               }
      {158 => Macaulay2Doc::changes, 1.7                                               }
      {159 => Macaulay2Doc::changes, 1.8                                               }
      {160 => Macaulay2Doc::changes, 1.9                                               }
      {161 => Macaulay2Doc::changes, 1.9.1                                             }
      {162 => Macaulay2Doc::changes, 1.10                                              }
      {163 => Macaulay2Doc::changes, 1.11                                              }
      {164 => Macaulay2Doc::changes, 1.12                                              }
      {165 => Macaulay2Doc::changes, 1.13                                              }
      {166 => Macaulay2Doc::complete                                                   }
      {167 => Macaulay2Doc::complete(ChainComplex)                                     }
      {168 => Macaulay2Doc::computing resolutions                                      }
      {169 => Macaulay2Doc::cone(ChainComplexMap)                                      }
      {170 => Macaulay2Doc::David Eisenbud                                             }
      {171 => Macaulay2Doc::dd                                                         }
      {172 => Macaulay2Doc::document                                                   }
      {173 => Macaulay2Doc::document(..., Inputs => ...)                               }
      {174 => Macaulay2Doc::document(..., Key => ...)                                  }
      {175 => Macaulay2Doc::document(..., Outputs => ...)                              }
      {176 => Macaulay2Doc::documentation keys                                         }
      {177 => Macaulay2Doc::eagonNorthcott(Matrix)                                     }
      {178 => Macaulay2Doc::Elementary uses of Groebner bases I. Math 634 Fall 2005    }
      {179 => Macaulay2Doc::engine                                                     }
      {180 => Macaulay2Doc::Ext^ZZ(CoherentSheaf,CoherentSheaf)                        }
      {181 => Macaulay2Doc::Ext^ZZ(CoherentSheaf,SumOfTwists)                          }
      {182 => Macaulay2Doc::Ext^ZZ(Matrix,Module)                                      }
      {183 => Macaulay2Doc::Ext^ZZ(Module,Matrix)                                      }
      {184 => Macaulay2Doc::Ext^ZZ(Module,Module)                                      }
      {185 => Macaulay2Doc::extend(ChainComplex,ChainComplex,Matrix)                   }
      {186 => Macaulay2Doc::extracting information from chain complexes                }
      {187 => Macaulay2Doc::factor                                                     }
      {188 => Macaulay2Doc::FastNonminimal                                             }
      {189 => Macaulay2Doc::findSynonyms(Symbol)                                       }
      {190 => Macaulay2Doc::frac                                                       }
      {191 => Macaulay2Doc::free resolutions of modules                                }
      {192 => Macaulay2Doc::gbTrace                                                    }
      {193 => Macaulay2Doc::getNonUnit                                                 }
      {194 => Macaulay2Doc::graded and multigraded polynomial rings                    }
      {195 => Macaulay2Doc::heft                                                       }
      {196 => Macaulay2Doc::HH^ZZ CoherentSheaf                                        }
      {197 => Macaulay2Doc::Hilbert functions and free resolutions                     }
      {198 => Macaulay2Doc::Hom(Module,ChainComplex)                                   }
      {199 => Macaulay2Doc::Ideal                                                      }
      {200 => Macaulay2Doc::ideals                                                     }
      {201 => Macaulay2Doc::ideals to and from modules                                 }
      {202 => Macaulay2Doc::LengthLimit                                                }
      {203 => Macaulay2Doc::lift                                                       }
      {204 => Macaulay2Doc::maps between chain complexes                               }
      {205 => Macaulay2Doc::methods                                                    }
      {206 => Macaulay2Doc::minimalBetti(Ideal)                                        }
      {207 => Macaulay2Doc::Module                                                     }
      {208 => Macaulay2Doc::modules                                                    }
      {209 => Macaulay2Doc::modules in Macaulay2                                       }
      {210 => Macaulay2Doc::nullhomotopy                                               }
      {211 => Macaulay2Doc::packages provided with Macaulay2                           }
      {212 => Macaulay2Doc::path                                                       }
      {213 => Macaulay2Doc::pdim                                                       }
      {214 => Macaulay2Doc::peek'(ZZ,Thing)                                            }
      {215 => Macaulay2Doc::pseudocode                                                 }
      {216 => Macaulay2Doc::regularity                                                 }
      {217 => Macaulay2Doc::replacements for commands and scripts from Macaulay        }
      {218 => Macaulay2Doc::Resolution                                                 }
      {219 => Macaulay2Doc::resolution                                                 }
      {220 => Macaulay2Doc::resolution(..., DegreeLimit => ...)                        }
      {221 => Macaulay2Doc::resolution(..., FastNonminimal => ...)                     }
      {222 => Macaulay2Doc::resolution(..., HardDegreeLimit => ...)                    }
      {223 => Macaulay2Doc::resolution(..., LengthLimit => ...)                        }
      {224 => Macaulay2Doc::resolution(..., PairLimit => ...)                          }
      {225 => Macaulay2Doc::resolution(..., SortStrategy => ...)                       }
      {226 => Macaulay2Doc::resolution(..., StopBeforeComputation => ...)              }
      {227 => Macaulay2Doc::resolution(..., Strategy => ...)                           }
      {228 => Macaulay2Doc::resolution(..., SyzygyLimit => ...)                        }
      {229 => Macaulay2Doc::resolution(Ideal)                                          }
      {230 => Macaulay2Doc::resolution(Matrix)                                         }
      {231 => Macaulay2Doc::resolution(Module)                                         }
      {232 => Macaulay2Doc::resolution(MonomialIdeal)                                  }
      {233 => Macaulay2Doc::RingMap RingElement                                        }
      {234 => Macaulay2Doc::running Macaulay2 in emacs                                 }
      {235 => Macaulay2Doc::Schreyer orders                                            }
      {236 => Macaulay2Doc::Singular Book 2.4.12                                       }
      {237 => Macaulay2Doc::Singular Book 2.4.15                                       }
      {238 => Macaulay2Doc::Singular Book 2.5.18                                       }
      {239 => Macaulay2Doc::source(ChainComplexMap)                                    }
      {240 => Macaulay2Doc::status                                                     }
      {241 => Macaulay2Doc::syzygyScheme                                               }
      {242 => Macaulay2Doc::target(ChainComplexMap)                                    }
      {243 => Macaulay2Doc::Tutorial: Canonical Embeddings of Plane Curves and Gonality}
      {244 => Macaulay2Doc::Tutorial: Elementary uses of Groebner bases                }
      {245 => Macaulay2Doc::Tutorial: Fano varieties                                   }
      {246 => Macaulay2Doc::two dimensional formatting                                 }
      {247 => Macaulay2Doc::Usage                                                      }
      {248 => Macaulay2Doc::using Macaulay2 with emacs after it has been set up        }
      {249 => MCMApproximations::approximation                                         }
      {250 => MCMApproximations::MCMApproximations                                     }
      {251 => MCMApproximations::syzygyModule                                          }
      {252 => MultiplierIdealsDim2::MultiplierIdealsDim2                               }
      {253 => MultiplierIdealsDim2::RelativeCanonicalDivisor                           }
      {254 => NCAlgebra::Basic operations on noncommutative algebras                   }
      {255 => NCAlgebra::betti(NCChainComplex)                                         }
      {256 => NCAlgebra::NCChainComplex                                                }
      {257 => NCAlgebra::NCMatrix // NCMatrix                                          }
      {258 => NCAlgebra::resolution(NCMatrix)                                          }
      {259 => NCAlgebra::rightKernel                                                   }
      {260 => NCAlgebra::Using the Bergman interface                                   }
      {261 => NonminimalComplexes::SVDBetti                                            }
      {262 => NormalToricVarieties::makeSmooth(NormalToricVariety)                     }
      {263 => NormalToricVarieties::Making normal toric varieties                      }
      {264 => NormalToricVarieties::Resolution of singularities                        }
      {265 => PieriMaps::pieri                                                         }
      {266 => PieriMaps::PieriMaps                                                     }
      {267 => PieriMaps::pureFree                                                      }
      {268 => Points::expectedBetti                                                    }
      {269 => Points::minMaxResolution                                                 }
      {270 => Points::Points                                                           }
      {271 => Posets::Example: Constructing common posets                              }
      {272 => Posets::Example: Hibi ideals                                             }
      {273 => Posets::Example: LCM-lattices                                            }
      {274 => Posets::lcmLattice                                                       }
      {275 => Posets::resolutionPoset                                                  }
      {276 => Posets::resolutionPoset(ChainComplex)                                    }
      {277 => Posets::resolutionPoset(Ideal)                                           }
      {278 => Posets::resolutionPoset(MonomialIdeal)                                   }
      {279 => PrimaryDecomposition::primaryDecomposition(..., Strategy => ...)         }
      {280 => PruneComplex::PruneComplex                                               }
      {281 => PruneComplex::pruneComplex                                               }
      {282 => PruneComplex::toChainComplex                                             }
      {283 => RandomMonomialIdeals::bettiStats                                         }
      {284 => RandomSpaceCurves::knownUnirationalComponentOfSpaceCurves                }
      {285 => RandomSpaceCurves::spaceCurve                                            }
      {286 => ReesAlgebra::PlaneCurveSingularities                                     }
      {287 => ReesAlgebra::ReesAlgebra                                                 }
      {288 => ReesAlgebra::reesIdeal                                                   }
      {289 => Regularity::Regularity                                                   }
      {290 => RelativeCanonicalResolution::eagonNorthcottType                          }
      {291 => RelativeCanonicalResolution::iteratedMC                                  }
      {292 => RelativeCanonicalResolution::liftMatrixToENT                             }
      {293 => RelativeCanonicalResolution::RelativeCanonicalResolution                 }
      {294 => RelativeCanonicalResolution::resCurveOnScroll                            }
      {295 => RelativeCanonicalResolution::rkSyzModules                                }
      {296 => SchurRings::schurResolution                                              }
      {297 => SchurRings::schurResolution(..., DegreeLimit => ...)                     }
      {298 => SchurRings::schurResolution(..., SyzygyLimit => ...)                     }
      {299 => SchurRings::SchurRings                                                   }
      {300 => SimplicialComplexes::lyubeznikComplex                                    }
      {301 => SimplicialComplexes::superficialComplex                                  }
      {302 => SLnEquivariantMatrices::SLnEquivariantMatrices                           }
      {303 => SpectralSequences::basis(List,SpectralSequencePage)                      }
      {304 => SpectralSequences::hilbertPolynomial(SpectralSequencePage)               }
      {305 => SpectralSequences::minimalPresentation(SpectralSequence)                 }
      {306 => SpectralSequences::minimalPresentation(SpectralSequencePage)             }
      {307 => SpectralSequences::Page                                                  }
      {308 => SpectralSequences::PageMap                                               }
      {309 => SpectralSequences::pruningMaps                                           }
      {310 => SpectralSequences::pruningMaps(SpectralSequencePage)                     }
      {311 => SpectralSequences::Spectral sequences and hypercohomology calculations   }
      {312 => SpectralSequences::SpectralSequence ^ ZZ                                 }
      {313 => SpectralSequences::SpectralSequence _ ZZ                                 }
      {314 => SpectralSequences::spectralSequencePage(FilteredComplex,ZZ)              }
      {315 => SpectralSequences::SpectralSequencePageMap ^ List                        }
      {316 => SpectralSequences::SpectralSequencePageMap _ List                        }
      {317 => TateOnProducts::beilinson                                                }
      {318 => TateOnProducts::beilinsonWindow                                          }
      {319 => TateOnProducts::coarseMultigradedRegularity                              }
      {320 => TateOnProducts::cohomologyHashTable                                      }
      {321 => TateOnProducts::cohomologyMatrix                                         }
      {322 => TateOnProducts::composedFunctions                                        }
      {323 => TateOnProducts::cornerComplex                                            }
      {324 => TateOnProducts::eulerPolynomialTable                                     }
      {325 => TateOnProducts::firstQuadrantComplex                                     }
      {326 => TateOnProducts::lastQuadrantComplex                                      }
      {327 => TateOnProducts::strand                                                   }
      {328 => TateOnProducts::tateExtension                                            }
      {329 => TateOnProducts::TateOnProducts                                           }
      {330 => TateOnProducts::tateResolution                                           }
      {331 => TensorComplexes::flattenedESTensor                                       }
      {332 => TensorComplexes::pureResES                                               }
      {333 => TensorComplexes::pureResES1                                              }
      {334 => TensorComplexes::pureResTC                                               }
      {335 => TensorComplexes::pureResTC1                                              }
      {336 => TensorComplexes::tensorComplex1                                          }
      {337 => TensorComplexes::TensorComplexes                                         }
      {338 => TestIdeals::isCohenMacaulay                                              }
      {339 => Text::html(TEX)                                                          }
      {340 => Text::MarkUpTypeWithOptions                                              }
      {341 => TorAlgebra::TorAlgebra                                                   }
      {342 => Triplets::Triplet                                                        }
      {343 => Triplets::Triplets                                                       }
      {344 => VectorFields::isFreeDivisor                                              }
      {345 => VersalDeformations::versalDeformation                                    }
      {346 => VirtualResolutions::isVirtual                                            }
      {347 => VirtualResolutions::isVirtual(..., Strategy => ...)                      }
      {348 => VirtualResolutions::resolveViaFatPoint                                   }
      {349 => VirtualResolutions::virtualOfPair                                        }
      {350 => VirtualResolutions::VirtualResolutions                                   }

o73 : NumberedVerticalList

i74 : R

o74 = R

o74 : PolynomialRing

i75 : describe R

o75 = QQ[x, Degrees => {1}, Heft => {1}, MonomialOrder => {MonomialSize => 32}, DegreeRank => 1]
                                                          {GRevLex => {1}    }
                                                          {Position => Up    }

i76 : peek R

o76 = PolynomialRing of RingElement{(?, R, R) => -*Function[/Applications/Macaulay2-1.14/share/Macaulay2/Core/enginering.m2:170:21-170:36]*-                                                                                                     }
                                    (_, R, monoid[x, Degrees => {1}, Heft => {1}, MonomialOrder => {MonomialSize => 32}, DegreeRank => 1]) => -*Function[/Applications/Macaulay2-1.14/share/Macaulay2/Core/orderedmonoidrings.m2:226:27-226:78]*-
                                                                                                   {GRevLex => {1}    }
                                                                                                   {Position => Up    }
                                    (lift, List, R, QQ) => -*Function[/Applications/Macaulay2-1.14/share/Macaulay2/Core/enginering.m2:195:44-195:65]*-
                                    (lift, List, R, R) => -*Function[/Applications/Macaulay2-1.14/share/Macaulay2/Core/enginering.m2:159:29-159:43]*-
                                    (lift, List, R, ZZ) => -*Function[/Applications/Macaulay2-1.14/share/Macaulay2/Core/enginering.m2:216:50-216:75]*-
                                    (lift, Matrix, R, QQ) => -*Function[/Applications/Macaulay2-1.14/share/Macaulay2/Core/enginering.m2:191:46-191:87]*-
                                    (lift, Matrix, R, R) => -*Function[/Applications/Macaulay2-1.14/share/Macaulay2/Core/enginering.m2:160:31-160:45]*-
                                    (lift, Matrix, R, ZZ) => -*Function[/Applications/Macaulay2-1.14/share/Macaulay2/Core/enginering.m2:212:49-212:91]*-
                                    (lift, Module, R, QQ) => -*Function[/Applications/Macaulay2-1.14/share/Macaulay2/Core/enginering.m2:189:46-189:80]*-
                                    (lift, Module, R, R) => -*Function[/Applications/Macaulay2-1.14/share/Macaulay2/Core/enginering.m2:162:31-162:45]*-
                                    (lift, Module, R, ZZ) => -*Function[/Applications/Macaulay2-1.14/share/Macaulay2/Core/enginering.m2:210:49-210:84]*-
                                    (lift, MutableMatrix, R, QQ) => -*Function[/Applications/Macaulay2-1.14/share/Macaulay2/Core/enginering.m2:193:53-193:99]*-
                                    (lift, MutableMatrix, R, R) => -*Function[/Applications/Macaulay2-1.14/share/Macaulay2/Core/enginering.m2:163:38-163:52]*-
                                    (lift, MutableMatrix, R, ZZ) => -*Function[/Applications/Macaulay2-1.14/share/Macaulay2/Core/enginering.m2:214:56-214:105]*-
                                    (lift, R, QQ) => -*Function[/Applications/Macaulay2-1.14/share/Macaulay2/Core/enginering.m2:187:39-187:75]*-
                                    (lift, R, R) => -*Function[/Applications/Macaulay2-1.14/share/Macaulay2/Core/enginering.m2:155:24-155:36]*-
                                    (lift, R, ZZ) => -*Function[/Applications/Macaulay2-1.14/share/Macaulay2/Core/enginering.m2:208:39-208:74]*-
                                    (promote, List, QQ, R) => -*Function[/Applications/Macaulay2-1.14/share/Macaulay2/Core/enginering.m2:194:50-194:62]*-
                                    (promote, List, R, R) => -*Function[/Applications/Macaulay2-1.14/share/Macaulay2/Core/enginering.m2:157:35-157:38]*-
                                    (promote, List, ZZ, R) => -*Function[/Applications/Macaulay2-1.14/share/Macaulay2/Core/enginering.m2:215:50-215:78]*-
                                    (promote, Matrix, QQ, R) => -*Function[/Applications/Macaulay2-1.14/share/Macaulay2/Core/enginering.m2:190:52-190:78]*-
                                    (promote, Matrix, R, R) => -*Function[/Applications/Macaulay2-1.14/share/Macaulay2/Core/enginering.m2:158:37-158:40]*-
                                    (promote, Matrix, ZZ, R) => -*Function[/Applications/Macaulay2-1.14/share/Macaulay2/Core/enginering.m2:211:52-211:79]*-
                                    (promote, Module, QQ, R) => -*Function[/Applications/Macaulay2-1.14/share/Macaulay2/Core/enginering.m2:188:52-188:78]*-
                                    (promote, Module, R, R) => -*Function[/Applications/Macaulay2-1.14/share/Macaulay2/Core/enginering.m2:161:37-161:40]*-
                                    (promote, Module, ZZ, R) => -*Function[/Applications/Macaulay2-1.14/share/Macaulay2/Core/enginering.m2:209:52-209:79]*-
                                    (promote, MutableMatrix, QQ, R) => -*Function[/Applications/Macaulay2-1.14/share/Macaulay2/Core/enginering.m2:192:59-192:90]*-
                                    (promote, MutableMatrix, R, R) => -*Function[/Applications/Macaulay2-1.14/share/Macaulay2/Core/enginering.m2:164:44-164:47]*-
                                    (promote, MutableMatrix, ZZ, R) => -*Function[/Applications/Macaulay2-1.14/share/Macaulay2/Core/enginering.m2:213:59-213:93]*-
                                    (promote, QQ, R) => -*Function[/Applications/Macaulay2-1.14/share/Macaulay2/Core/enginering.m2:185:37-185:71]*-
                                    (promote, R, R) => -*Function[/Applications/Macaulay2-1.14/share/Macaulay2/Core/enginering.m2:156:28-156:31]*-
                                    (promote, ZZ, R) => -*Function[/Applications/Macaulay2-1.14/share/Macaulay2/Core/enginering.m2:206:37-206:71]*-
                                    0 => 0
                                    1 => 1
                                    basering => QQ
                                    baseRings => {ZZ, QQ}
                                    char => 0
                                    degreesMonoid => monoid[T, Degrees => {1}, MonomialOrder => {MonomialSize => 32}, DegreeRank => 1, Inverses => true, Global => false]
                                                                                                {Weights => {-1}   }
                                                                                                {GroupLex => 1     }
                                                                                                {Position => Up    }
                                    degreesRing => ZZ[T]
                                    Engine => true
                                    expression => -*Function[/Applications/Macaulay2-1.14/share/Macaulay2/Core/orderedmonoidrings.m2:232:35-239:43]*-
                                    factor => -*Function[/Applications/Macaulay2-1.14/share/Macaulay2/Core/orderedmonoidrings.m2:253:29-283:76]*-
                                    FlatMonoid => monoid[x, Degrees => {1}, Heft => {1}, MonomialOrder => {MonomialSize => 32}, DegreeRank => 1]
                                                                                                          {GRevLex => {1}    }
                                                                                                          {Position => Up    }
                                    generatorExpressions => {x}
                                    generators => {x}
                                    generatorSymbols => {x}
                                    indexStrings => HashTable{x => x}
                                    indexSymbols => HashTable{x => x}
                                    isCommutative => true
                                    isPrime => -*Function[/Applications/Macaulay2-1.14/share/Macaulay2/Core/orderedmonoidrings.m2:284:27-288:22]*-
                                    liftDegree => -*Function[/Applications/Macaulay2-1.14/share/Macaulay2/Core/enginering.m2:59:20-61:38]*-
                                    monoid => monoid[x, Degrees => {1}, Heft => {1}, MonomialOrder => {MonomialSize => 32}, DegreeRank => 1]
                                                                                                      {GRevLex => {1}    }
                                                                                                      {Position => Up    }
                                    numallvars => 1
                                    promoteDegree => -*Function[/Applications/Macaulay2-1.14/share/Macaulay2/Core/enginering.m2:59:20-61:38]*-
                                    raw creation log => -*a bagged function application expression*-
                                    RawRing => QQGMP[x,
                                                 DegreeLength => 1,
                                                 Degrees => {1},
                                                 Heft => {1},
                                                 MonomialOrder => {
                                                   GRevLex => {1},
                                                   Position => Up
                                                   }
                                                 ]

i77 : f = () -> ( QQ [ local y _ 0 ] )

o77 = f

o77 : FunctionClosure

i78 : f()
warning: clearing value of symbol y to allow access to subscripted variables based on it
       : debug with expression   debug 9371   or with command line option   --debug 9371

o78 = QQ[y ]
          0

o78 : PolynomialRing

i79 : f()
warning: clearing value of symbol y to allow access to subscripted variables based on it
       : debug with expression   debug 9371   or with command line option   --debug 9371

o79 = QQ[y ]
          0

o79 : PolynomialRing

i80 : y

o80 = y

o80 : IndexedVariableTable

i81 : peek y

o81 = IndexedVariableTable{0 => y      }
                                 0
                           symbol$ => y

i82 : debug 9371

o82 = 9371

i83 : y=0

o83 = 0

i84 : f()
stdio:77:13:(3):[1]: error: clearing value of symbol y to allow access to subscripted variables based on it

i85 : restart
Macaulay2, version 1.14
--loading configuration for package "FourTiTwo" from file /Users/dan/Library/Application Support/Macaulay2/init-FourTiTwo.m2
--loading configuration for package "Topcom" from file /Users/dan/Library/Application Support/Macaulay2/init-Topcom.m2
with packages: ConwayPolynomials, Elimination, IntegralClosure, InverseSystems, LLLBases, PrimaryDecomposition,
               ReesAlgebra, TangentCone, Truncations

i1 : f = () -> ( QQ [ local y _ 0 ] )

o1 = f

o1 : FunctionClosure

i2 : QQ[y]

o2 = QQ[y]

o2 : PolynomialRing

i3 : f()
warning: clearing value of symbol y to allow access to subscripted variables based on it
       : debug with expression   debug 9371   or with command line option   --debug 9371

o3 = QQ[y ]
         0

o3 : PolynomialRing

i4 : debug 9371

o4 = 9371

i5 : f()
stdio:1:13:(3):[1]: error: clearing value of symbol y to allow access to subscripted variables based on it

i6 : help

o6 = initial help
     ************

     Welcome to Macaulay2


     Try entering '2+2' at your next input prompt, which begins with i.  The two output prompts begin with o.
     The first one, with the equal sign, '=', gives the value computed from your input, and the second one,
     with the colon, ':', tells what type of thing the value is.


     Type one of these commands to get started reading the documentation:
       * copyright                         -- the copyright
       * help "Macaulay2"                  -- top node of the documentation.
       * help "reading the documentation"  -- 
       * help "getting started"            -- 
       * help "a first Macaulay2 session"  -- 
       * help x                            -- show documentation for x
       * help about x                      -- show documentation about x
       * help about (x,Body=>true)         -- show documentation mentioning x
       * ? f                               -- display brief documentation for a function f
       * printWidth = 80                   -- set print width to 80 characters
       * viewHelp                          -- view documentation in a browser
       * viewHelp x                        -- view documentation on x in browser
     To read the documentation in info form, in case you happen to be running Macaulay2 in a terminal window,
     replace "help" by "infoHelp" in any of the commands above.

o6 : DIV

i7 : 

Process M2 finished
Clone this wiki locally